LSAT and Law School Admissions Forum

Get expert LSAT preparation and law school admissions advice from PowerScore Test Preparation.

User avatar
 Dave Killoran
PowerScore Staff
  • PowerScore Staff
  • Posts: 5853
  • Joined: Mar 25, 2011
|
#43452
Complete Question Explanation
(The complete setup for this game can be found here: lsat/viewtopic.php?t=16410)

The correct answer choice is (A)

For this question, the correct answer choice will be the one that is Not Necessarily True, and the four incorrect answer choices all Must be True.

If P visits Vancouver, S cannot visit Vancouver, but still has the options of visiting either Montreal or Toronto. Since S would not necessarily have to go to Montreal, answer choice (A) is the correct answer choice.

Answer choice (B) is dictated by the fourth and second rules, respectively, so it must be true, and it is therefore incorrect. Answer choice (C) must be true as discussed in the game setup, and therefore answer choice (C) is incorrect. Answer choices (D) and (E) are also proven as true by the discussion of the 3-2 distribution, and therefore both answers are incorrect.

Get the most out of your LSAT Prep Plus subscription.

Analyze and track your performance with our Testing and Analytics Package.